Question

1. Consider the following production function: Y=F(A,L,K)=A(K^α)(L^(1-α)) where α < 1. a. Derive the Marginal Product...

1. Consider the following production function:

Y=F(A,L,K)=A(K^α)(L^(1-α))

where α < 1.

a. Derive the Marginal Product of Labor(MPL).

b. Show that this production function

exhibit diminishing MPL.

c. Derive the Marginal Production of Technology (MPA).

d. Does this production function exhibit diminishing MPA? Prove or disprove

Homework Answers

Answer #1

(a) Y= F(A,L,K) = A Ka L1-a

For calculating MPL , take the first partial derivative of Y with respect to L, we get:

MPL = (1-a) A Ka L1-a-1 = (1-a) AKa L-a.

(b) To see whether the production function exhibit diminishing MPL or not , take the second order partial derivative with respect to L, we get:
= (-a)(1-a) AKa L-a-1 < 0   

Hence, It shows that the production function exhibit diminishing returns to factor L.

(c) To calculate MPA , take the partial derivative of Y with respect to A , we get:
MPA = Ka L1-a

(d) To see whether the production function exhibit diminishing MPA or not, take the second order partial derivative of Y with respect to A, we get:

= 0 .

This implies that the production function exhibits constant returns to factor A.

Know the answer?
Your Answer:

Post as a guest

Your Name:

What's your source?

Earn Coins

Coins can be redeemed for fabulous gifts.

Not the answer you're looking for?
Ask your own homework help question
Similar Questions
Consider the following production function: y = F(K, L, D) = TK^αL^β/D^α+β−1 where K, L and...
Consider the following production function: y = F(K, L, D) = TK^αL^β/D^α+β−1 where K, L and D represent capital, labor and land inputs respectively. Denote by s the capital-labor ratio (s = K L ). T captures technological progress and is assumed constant here. α and β are two parameters. (a) (2.5 marks) Does y exhibits constant returns to scale? Show your work. (b) (2.5 marks) Find the marginal product of capital (MPK), the marginal product of labor (MP L),...
(a) Show that the following Cobb-Douglas production function, f(K,L) = KαL1−α, has constant returns to scale....
(a) Show that the following Cobb-Douglas production function, f(K,L) = KαL1−α, has constant returns to scale. (b) Derive the marginal products of labor and capital. Show that you the MPL is decreasing on L and that the MPK is decreasing in K.
Consider the production function Q = f(L,K) = 10KL / K+L. The marginal products of labor...
Consider the production function Q = f(L,K) = 10KL / K+L. The marginal products of labor and capital for this function are given by MPL = 10K^2 / (K +L)^2, MPK = 10L^2 / (K +L)^2. (a) In the short run, assume that capital is fixed at K = 4. What is the production function for the firm (quantity as a function of labor only)? What are the average and marginal products of labor? Draw APL and MPL on one...
Consider the following production function Y=z*(a*K + (1-a)*N) where z represents total factor productivity, a is...
Consider the following production function Y=z*(a*K + (1-a)*N) where z represents total factor productivity, a is a parameter between 0 and 1, K is the level of capital, and N is labor. We want to check if this function satisfies our basic assumptions about production functions. 1. Does this production function exhibit constant returns to scale? Ex- plain 2. Is the marginal product of labor always positive? Explain 3. Does this function exhibit diminishing marginal product of labor? Ex- plain...
Consider the Cobb-Douglas production function F (L, K) = (A)(L^α)(K^1/2) , where α > 0 and...
Consider the Cobb-Douglas production function F (L, K) = (A)(L^α)(K^1/2) , where α > 0 and A > 0. 1. The Cobb-Douglas function can be either increasing, decreasing or constant returns to scale depending on the values of the exponents on L and K. Prove your answers to the following three cases. (a) For what value(s) of α is F(L,K) decreasing returns to scale? (b) For what value(s) of α is F(L,K) increasing returns to scale? (c) For what value(s)...
A firm’s production function is given by Q = 5K1/3 + 10L1/3, where K and L...
A firm’s production function is given by Q = 5K1/3 + 10L1/3, where K and L denote quantities of capital and labor, respectively. Derive expressions (formulas) for the marginal product of each input. Does more of each input increase output? Does each input exhibit diminishing marginal returns? Prove. Derive an expression for the marginal rate of technical substitution (MRTS) of labor for capital. Suppose the price of capital, r = 1, and the price of labor, w = 1.   The...
Consider the production function Y = F (K, L) = Ka * L1-a, where 0 <...
Consider the production function Y = F (K, L) = Ka * L1-a, where 0 < α < 1. The national saving rate is s, the labor force grows at a rate n, and capital depreciates at rate δ. (a) Show that F has constant returns to scale. (b) What is the per-worker production function, y = f(k)? (c) Solve for the steady-state level of capital per worker (in terms of the parameters of the model). (d) Solve for the...
Derive the cost function from the following CES production function y = A *(α*L^p+ (1-α)*K^ρ)^(1/ρ)
Derive the cost function from the following CES production function y = A *(α*L^p+ (1-α)*K^ρ)^(1/ρ)
Consider a production function for an economy: Y = 20(L.5K.4N.1)where L is labor, K is capital,...
Consider a production function for an economy: Y = 20(L.5K.4N.1)where L is labor, K is capital, and N is land. In this economy the factors of production are in fixed supply with L = 100, K = 100, and N = 100. a) What is the level of output in this country? b) Does this production function exhibit constant returns to scale? Demonstrate by an example. c) If the economy is competitive so that factors of production are paid the...
A firm’s production function is ? = ?Lα ?β where A, α, and β are positive...
A firm’s production function is ? = ?Lα ?β where A, α, and β are positive constants. The firm currently uses 500 units of labor and 40 units of capital. If the firm adds 1 more unit of labor, what happens to productivity of capital? Explain. b. Given a production function Q = f(L, K), if marginal product of labor and marginal product of capital are both positive, then this function displays diminishing MRTS. Explain if this statement is true...
ADVERTISEMENT
Need Online Homework Help?

Get Answers For Free
Most questions answered within 1 hours.

Ask a Question
ADVERTISEMENT